Computing $sum_{n = 1}^{+ infty} frac{1}{n^2 + 1}$ using Fourier series.












1












$begingroup$



$f$ is a $2 pi$ periodic function on $ ] - pi,pi[$ defined as :



$$f(x) = e^{-x}$$



Using Fourier series, compute the sums :



$$ sum_{n = 1}^{+ infty} frac{1}{n^2 + 1} , sum_{- infty}^{+ infty} frac{1}{n^2 + 1} $$






How do I compute $sum_{n = 1}^{+ infty} frac{1}{n^2 + 1}$ ?



I have computed the fourier coefficient and found that:



$$a_0 = frac{2}{pi} (1 - e^{- pi}) $$



$$a_n = frac{2}{pi(n^2 + 1)} (1 - (-1)^n . e^{- pi} ) $$



$$b_n = frac{2n}{pi(n^2 + 1)} (1 - (-1)^n . e^{- pi} ) $$



Using Dirichlet theorem and taking $x = 0$ to make the $b_n$ disappear, I get:



$$S_f (1) = frac{2}{pi} (1 - e^{- pi}) + sum_{n = 1}^{+ infty} frac{2}{pi(n^2 + 1)} (1 - (-1)^n . e^{- pi} ) . (-1)^n = 1 $$



I do not see how to proceed to get the value of the sum?



The same problem for the second sum.










share|cite|improve this question









$endgroup$








  • 1




    $begingroup$
    "same problem for the second sum" - not exactly; once you find the first sum, multiply it by $2$ and add $1$.
    $endgroup$
    – user170231
    Jan 30 at 16:20










  • $begingroup$
    @user170231 Thank you. I'm still stuck with the first sun though.
    $endgroup$
    – Zouhair El Yaagoubi
    Jan 30 at 16:21






  • 1




    $begingroup$
    Your coefficients are not correct: $$a_0=frac1piint_{-pi}^pi e^{-x},mathrm dx=frac{e^pi-e^{-pi}}pi=frac{2sinhpi}pi$$ Similarly you should have ended up with $$a_n=frac{2(-1)^nsinhpi}{pi(n^2+1)}text{ and }b_n=frac{2n(-1)^nsinhpi}{pi(n^2+1)}$$ I think you are just missing a factor of $e^pi$ in certain places. (NB: $2sinhpi=e^pi-e^{-pi}$.)
    $endgroup$
    – user170231
    Jan 30 at 17:04










  • $begingroup$
    @user170231 To compute the coefficients, I used the formula: $$ a_n = frac{2}{pi} int_{0}^{pi} f(x) cos (nx)$$ and : $$ b_n = frac{2}{pi} int_{0}^{pi} f(x) sin (nx)$$ Isn't it correct?
    $endgroup$
    – Zouhair El Yaagoubi
    Jan 30 at 17:18








  • 1




    $begingroup$
    The formula I cited specifically works only for functions that are $2pi$-periodic over the interval $(-pi,pi)$. There is a formula for $P$-periodic functions over the interval $(x_0,x_0+P)$ listed on the page I linked under the section "A more general definition" (Eq. 4) for the more general case. It suggests$$a_n=frac2piint_0^pi f(x)cos nx,mathrm dx$$works for a function $f(x)$ that is $pi$-periodic over $(0,pi)$.
    $endgroup$
    – user170231
    Jan 30 at 18:33
















1












$begingroup$



$f$ is a $2 pi$ periodic function on $ ] - pi,pi[$ defined as :



$$f(x) = e^{-x}$$



Using Fourier series, compute the sums :



$$ sum_{n = 1}^{+ infty} frac{1}{n^2 + 1} , sum_{- infty}^{+ infty} frac{1}{n^2 + 1} $$






How do I compute $sum_{n = 1}^{+ infty} frac{1}{n^2 + 1}$ ?



I have computed the fourier coefficient and found that:



$$a_0 = frac{2}{pi} (1 - e^{- pi}) $$



$$a_n = frac{2}{pi(n^2 + 1)} (1 - (-1)^n . e^{- pi} ) $$



$$b_n = frac{2n}{pi(n^2 + 1)} (1 - (-1)^n . e^{- pi} ) $$



Using Dirichlet theorem and taking $x = 0$ to make the $b_n$ disappear, I get:



$$S_f (1) = frac{2}{pi} (1 - e^{- pi}) + sum_{n = 1}^{+ infty} frac{2}{pi(n^2 + 1)} (1 - (-1)^n . e^{- pi} ) . (-1)^n = 1 $$



I do not see how to proceed to get the value of the sum?



The same problem for the second sum.










share|cite|improve this question









$endgroup$








  • 1




    $begingroup$
    "same problem for the second sum" - not exactly; once you find the first sum, multiply it by $2$ and add $1$.
    $endgroup$
    – user170231
    Jan 30 at 16:20










  • $begingroup$
    @user170231 Thank you. I'm still stuck with the first sun though.
    $endgroup$
    – Zouhair El Yaagoubi
    Jan 30 at 16:21






  • 1




    $begingroup$
    Your coefficients are not correct: $$a_0=frac1piint_{-pi}^pi e^{-x},mathrm dx=frac{e^pi-e^{-pi}}pi=frac{2sinhpi}pi$$ Similarly you should have ended up with $$a_n=frac{2(-1)^nsinhpi}{pi(n^2+1)}text{ and }b_n=frac{2n(-1)^nsinhpi}{pi(n^2+1)}$$ I think you are just missing a factor of $e^pi$ in certain places. (NB: $2sinhpi=e^pi-e^{-pi}$.)
    $endgroup$
    – user170231
    Jan 30 at 17:04










  • $begingroup$
    @user170231 To compute the coefficients, I used the formula: $$ a_n = frac{2}{pi} int_{0}^{pi} f(x) cos (nx)$$ and : $$ b_n = frac{2}{pi} int_{0}^{pi} f(x) sin (nx)$$ Isn't it correct?
    $endgroup$
    – Zouhair El Yaagoubi
    Jan 30 at 17:18








  • 1




    $begingroup$
    The formula I cited specifically works only for functions that are $2pi$-periodic over the interval $(-pi,pi)$. There is a formula for $P$-periodic functions over the interval $(x_0,x_0+P)$ listed on the page I linked under the section "A more general definition" (Eq. 4) for the more general case. It suggests$$a_n=frac2piint_0^pi f(x)cos nx,mathrm dx$$works for a function $f(x)$ that is $pi$-periodic over $(0,pi)$.
    $endgroup$
    – user170231
    Jan 30 at 18:33














1












1








1





$begingroup$



$f$ is a $2 pi$ periodic function on $ ] - pi,pi[$ defined as :



$$f(x) = e^{-x}$$



Using Fourier series, compute the sums :



$$ sum_{n = 1}^{+ infty} frac{1}{n^2 + 1} , sum_{- infty}^{+ infty} frac{1}{n^2 + 1} $$






How do I compute $sum_{n = 1}^{+ infty} frac{1}{n^2 + 1}$ ?



I have computed the fourier coefficient and found that:



$$a_0 = frac{2}{pi} (1 - e^{- pi}) $$



$$a_n = frac{2}{pi(n^2 + 1)} (1 - (-1)^n . e^{- pi} ) $$



$$b_n = frac{2n}{pi(n^2 + 1)} (1 - (-1)^n . e^{- pi} ) $$



Using Dirichlet theorem and taking $x = 0$ to make the $b_n$ disappear, I get:



$$S_f (1) = frac{2}{pi} (1 - e^{- pi}) + sum_{n = 1}^{+ infty} frac{2}{pi(n^2 + 1)} (1 - (-1)^n . e^{- pi} ) . (-1)^n = 1 $$



I do not see how to proceed to get the value of the sum?



The same problem for the second sum.










share|cite|improve this question









$endgroup$





$f$ is a $2 pi$ periodic function on $ ] - pi,pi[$ defined as :



$$f(x) = e^{-x}$$



Using Fourier series, compute the sums :



$$ sum_{n = 1}^{+ infty} frac{1}{n^2 + 1} , sum_{- infty}^{+ infty} frac{1}{n^2 + 1} $$






How do I compute $sum_{n = 1}^{+ infty} frac{1}{n^2 + 1}$ ?



I have computed the fourier coefficient and found that:



$$a_0 = frac{2}{pi} (1 - e^{- pi}) $$



$$a_n = frac{2}{pi(n^2 + 1)} (1 - (-1)^n . e^{- pi} ) $$



$$b_n = frac{2n}{pi(n^2 + 1)} (1 - (-1)^n . e^{- pi} ) $$



Using Dirichlet theorem and taking $x = 0$ to make the $b_n$ disappear, I get:



$$S_f (1) = frac{2}{pi} (1 - e^{- pi}) + sum_{n = 1}^{+ infty} frac{2}{pi(n^2 + 1)} (1 - (-1)^n . e^{- pi} ) . (-1)^n = 1 $$



I do not see how to proceed to get the value of the sum?



The same problem for the second sum.







real-analysis sequences-and-series fourier-analysis fourier-series






share|cite|improve this question













share|cite|improve this question











share|cite|improve this question




share|cite|improve this question










asked Jan 30 at 15:45









Zouhair El YaagoubiZouhair El Yaagoubi

538411




538411








  • 1




    $begingroup$
    "same problem for the second sum" - not exactly; once you find the first sum, multiply it by $2$ and add $1$.
    $endgroup$
    – user170231
    Jan 30 at 16:20










  • $begingroup$
    @user170231 Thank you. I'm still stuck with the first sun though.
    $endgroup$
    – Zouhair El Yaagoubi
    Jan 30 at 16:21






  • 1




    $begingroup$
    Your coefficients are not correct: $$a_0=frac1piint_{-pi}^pi e^{-x},mathrm dx=frac{e^pi-e^{-pi}}pi=frac{2sinhpi}pi$$ Similarly you should have ended up with $$a_n=frac{2(-1)^nsinhpi}{pi(n^2+1)}text{ and }b_n=frac{2n(-1)^nsinhpi}{pi(n^2+1)}$$ I think you are just missing a factor of $e^pi$ in certain places. (NB: $2sinhpi=e^pi-e^{-pi}$.)
    $endgroup$
    – user170231
    Jan 30 at 17:04










  • $begingroup$
    @user170231 To compute the coefficients, I used the formula: $$ a_n = frac{2}{pi} int_{0}^{pi} f(x) cos (nx)$$ and : $$ b_n = frac{2}{pi} int_{0}^{pi} f(x) sin (nx)$$ Isn't it correct?
    $endgroup$
    – Zouhair El Yaagoubi
    Jan 30 at 17:18








  • 1




    $begingroup$
    The formula I cited specifically works only for functions that are $2pi$-periodic over the interval $(-pi,pi)$. There is a formula for $P$-periodic functions over the interval $(x_0,x_0+P)$ listed on the page I linked under the section "A more general definition" (Eq. 4) for the more general case. It suggests$$a_n=frac2piint_0^pi f(x)cos nx,mathrm dx$$works for a function $f(x)$ that is $pi$-periodic over $(0,pi)$.
    $endgroup$
    – user170231
    Jan 30 at 18:33














  • 1




    $begingroup$
    "same problem for the second sum" - not exactly; once you find the first sum, multiply it by $2$ and add $1$.
    $endgroup$
    – user170231
    Jan 30 at 16:20










  • $begingroup$
    @user170231 Thank you. I'm still stuck with the first sun though.
    $endgroup$
    – Zouhair El Yaagoubi
    Jan 30 at 16:21






  • 1




    $begingroup$
    Your coefficients are not correct: $$a_0=frac1piint_{-pi}^pi e^{-x},mathrm dx=frac{e^pi-e^{-pi}}pi=frac{2sinhpi}pi$$ Similarly you should have ended up with $$a_n=frac{2(-1)^nsinhpi}{pi(n^2+1)}text{ and }b_n=frac{2n(-1)^nsinhpi}{pi(n^2+1)}$$ I think you are just missing a factor of $e^pi$ in certain places. (NB: $2sinhpi=e^pi-e^{-pi}$.)
    $endgroup$
    – user170231
    Jan 30 at 17:04










  • $begingroup$
    @user170231 To compute the coefficients, I used the formula: $$ a_n = frac{2}{pi} int_{0}^{pi} f(x) cos (nx)$$ and : $$ b_n = frac{2}{pi} int_{0}^{pi} f(x) sin (nx)$$ Isn't it correct?
    $endgroup$
    – Zouhair El Yaagoubi
    Jan 30 at 17:18








  • 1




    $begingroup$
    The formula I cited specifically works only for functions that are $2pi$-periodic over the interval $(-pi,pi)$. There is a formula for $P$-periodic functions over the interval $(x_0,x_0+P)$ listed on the page I linked under the section "A more general definition" (Eq. 4) for the more general case. It suggests$$a_n=frac2piint_0^pi f(x)cos nx,mathrm dx$$works for a function $f(x)$ that is $pi$-periodic over $(0,pi)$.
    $endgroup$
    – user170231
    Jan 30 at 18:33








1




1




$begingroup$
"same problem for the second sum" - not exactly; once you find the first sum, multiply it by $2$ and add $1$.
$endgroup$
– user170231
Jan 30 at 16:20




$begingroup$
"same problem for the second sum" - not exactly; once you find the first sum, multiply it by $2$ and add $1$.
$endgroup$
– user170231
Jan 30 at 16:20












$begingroup$
@user170231 Thank you. I'm still stuck with the first sun though.
$endgroup$
– Zouhair El Yaagoubi
Jan 30 at 16:21




$begingroup$
@user170231 Thank you. I'm still stuck with the first sun though.
$endgroup$
– Zouhair El Yaagoubi
Jan 30 at 16:21




1




1




$begingroup$
Your coefficients are not correct: $$a_0=frac1piint_{-pi}^pi e^{-x},mathrm dx=frac{e^pi-e^{-pi}}pi=frac{2sinhpi}pi$$ Similarly you should have ended up with $$a_n=frac{2(-1)^nsinhpi}{pi(n^2+1)}text{ and }b_n=frac{2n(-1)^nsinhpi}{pi(n^2+1)}$$ I think you are just missing a factor of $e^pi$ in certain places. (NB: $2sinhpi=e^pi-e^{-pi}$.)
$endgroup$
– user170231
Jan 30 at 17:04




$begingroup$
Your coefficients are not correct: $$a_0=frac1piint_{-pi}^pi e^{-x},mathrm dx=frac{e^pi-e^{-pi}}pi=frac{2sinhpi}pi$$ Similarly you should have ended up with $$a_n=frac{2(-1)^nsinhpi}{pi(n^2+1)}text{ and }b_n=frac{2n(-1)^nsinhpi}{pi(n^2+1)}$$ I think you are just missing a factor of $e^pi$ in certain places. (NB: $2sinhpi=e^pi-e^{-pi}$.)
$endgroup$
– user170231
Jan 30 at 17:04












$begingroup$
@user170231 To compute the coefficients, I used the formula: $$ a_n = frac{2}{pi} int_{0}^{pi} f(x) cos (nx)$$ and : $$ b_n = frac{2}{pi} int_{0}^{pi} f(x) sin (nx)$$ Isn't it correct?
$endgroup$
– Zouhair El Yaagoubi
Jan 30 at 17:18






$begingroup$
@user170231 To compute the coefficients, I used the formula: $$ a_n = frac{2}{pi} int_{0}^{pi} f(x) cos (nx)$$ and : $$ b_n = frac{2}{pi} int_{0}^{pi} f(x) sin (nx)$$ Isn't it correct?
$endgroup$
– Zouhair El Yaagoubi
Jan 30 at 17:18






1




1




$begingroup$
The formula I cited specifically works only for functions that are $2pi$-periodic over the interval $(-pi,pi)$. There is a formula for $P$-periodic functions over the interval $(x_0,x_0+P)$ listed on the page I linked under the section "A more general definition" (Eq. 4) for the more general case. It suggests$$a_n=frac2piint_0^pi f(x)cos nx,mathrm dx$$works for a function $f(x)$ that is $pi$-periodic over $(0,pi)$.
$endgroup$
– user170231
Jan 30 at 18:33




$begingroup$
The formula I cited specifically works only for functions that are $2pi$-periodic over the interval $(-pi,pi)$. There is a formula for $P$-periodic functions over the interval $(x_0,x_0+P)$ listed on the page I linked under the section "A more general definition" (Eq. 4) for the more general case. It suggests$$a_n=frac2piint_0^pi f(x)cos nx,mathrm dx$$works for a function $f(x)$ that is $pi$-periodic over $(0,pi)$.
$endgroup$
– user170231
Jan 30 at 18:33










3 Answers
3






active

oldest

votes


















3












$begingroup$

With the correct coefficients (see comments), you have for $x in]-pi,pi[$:



$$
e^{-x} = frac{2}{pi} sinh(pi) + 2 sinh(pi) sum_{n=1}^{infty}frac{ (-1)^n (nsin (nx) + cos(nx)) }{pi(n^2 + 1)}
$$



so



$$
e^{x} + e^{-x} = frac{4}{pi} sinh(pi) + 4 sinh(pi) sum_{n=1}^{infty}frac{ (-1)^n cos(nx) }{pi(n^2 + 1)}
$$



Now we can evaluate this for $x to pmpi$. To be more precise (see the answer by @kvantour), the evaluation $x to pmpi$ corresponds to the average of the two values at $pmpi$ which is exactly what the Fourier series converges to at this point of discontinuity.



$$
cosh (pm pi) = frac{2}{pi} sinh(pi)+ 2 sinh(pi) sum_{n=1}^{infty}frac{1 }{pi(n^2 + 1)}
$$



and in turn



$$
sum_{n=1}^{infty}frac{1 }{(n^2 + 1)} = frac{pi coth(pi) -1}{2 }
$$



Then the second question follows:
$$
sum_{- infty}^{+ infty} frac{1}{n^2 + 1} = 1 + 2 sum_{1}^{+ infty} frac{1}{n^2 + 1} = pi coth(pi) simeq 1.0037 ; pi
$$



Comment: for comparison,
$$
int_{- infty}^{+ infty} frac{1}{x^2 + 1} {rm{dx}} = pi
$$






share|cite|improve this answer











$endgroup$









  • 1




    $begingroup$
    You cannot make the statement $xrightarrowpi$ and just know its value. $f(x)$ is discontinuous in that point.
    $endgroup$
    – kvantour
    Jan 31 at 11:52








  • 1




    $begingroup$
    @kvantour Thank you. I have adressed that point and edited my answer accordingly.
    $endgroup$
    – Andreas
    Jan 31 at 15:13






  • 1




    $begingroup$
    Good point. done.
    $endgroup$
    – Andreas
    Jan 31 at 15:25



















1












$begingroup$

When you state Using Fourier series, it does not necessarily mean that you have to use the Fourier Cosine and Sine series. You can also write it in exponential form:




Any periodic function $tilde{f}(t)$ with period $2pi$ can be written
as:



$$ f(t) = sum_{n=-infty}^infty c_n,e^{int} $$ with $$
c_n=frac{1}{2pi}int_{-pi}^pi tilde f(t),e^{-int},textrm{d}t
$$

note: we make a distinction between the fourier series $f(t)$ and the periodic function $tilde f(t)$ to indicate that they are different when $tilde f(t)$ is discontiniuous.




When you apply this for $tilde f(t)$, which is periodic over $2pi$, discontinuous and defined in the region $]-pi,pi[$ as $exp(-t)$, you obtain



$$
c_n = frac{sinh(pi)}picdotfrac{(-1)^n,(1-in)}{1+n^2}
$$



giving you:



$$
f(t)=frac{sinh(pi)}pi sum_{n=-infty}^inftyfrac{(-1)^n,(1-in)}{1+n^2},e^{int}
$$



You already notice the familiar part in the sum which is of interest, the problem is the $(-1)^n$. This we can get rid of with the proper choice of $t$. If $t=pmpi$ then $exp(pm inpi)=(-1)^n$. But be advised $f(t)$ is periodic with a period of $2pi$ and is not continuous in the points $t=npi$. The value the Fourier Series will return is:



$$ f(pi)=f(-pi)= frac{sinh(pi)}pi sum_{n=-infty}^inftyfrac{(1-in)}{1+n^2} = frac{sinh(pi)}pi sum_{n=-infty}^inftyfrac{1}{1+n^2}$$



The latter reduction of the sum is straightforward as the imaginary part is zero.



But we cannot use this, as we do not know what $f(pi)$ is since $tilde f(t)$ is discontinuous at these points. Luckily, some smart people solved this conundrum and showed that at a discontinuity, the Fourier series converges to the average of the two values, i.e.



$$ f(pi)=f(-pi)=frac{lim_{trightarrowpi^-}tilde f(t) + lim_{trightarrow pi^+}tilde f(t)}{2} = frac{exp(-pi)+exp(pi)}{2} = cosh(pi)$$



See: Fourier series at discontinuities



So in the end, we have the solution:
$$f(pi)=cosh(pi)=frac{sinh(pi)}pi sum_{n=-infty}^inftyfrac{1}{1+n^2}$$
or



$$bbox[5px,border:2px solid #00A000]{pi coth(pi)=sum_{n=-infty}^inftyfrac{1}{1+n^2}=1+2sum_{n=1}^inftyfrac{1}{1+n^2}}$$



remark: computing $f(0)$ gives directly, without any fuss:



$$bbox[5px,border:2px solid #000000]{frac{pi}{sinh(pi)}=sum_{n=-infty}^inftyfrac{(-1)^n}{1+n^2}}$$






share|cite|improve this answer











$endgroup$









  • 1




    $begingroup$
    I computed the sum using $f(0)$, but I will pay more attention to the points of discontinuity, so thank you so much for your additional important information.
    $endgroup$
    – Zouhair El Yaagoubi
    Jan 31 at 12:11



















-2












$begingroup$

Try using Parseval's theorem,



$$frac{1}{pi}int_{-pi}^pi |f(x)|^2 dx = frac{a_0}{2} + sum_{n=1}^{infty} a_n^2+b_n^2$$



If it comes out too nasty, then I think your coeffecients might be off.






share|cite|improve this answer









$endgroup$













  • $begingroup$
    I do not see how Parseval's theorem will give me the sum, can you elaborate more? Although, I have checked the coefficient well. They seem to be correct.
    $endgroup$
    – Zouhair El Yaagoubi
    Jan 30 at 16:12












Your Answer





StackExchange.ifUsing("editor", function () {
return StackExchange.using("mathjaxEditing", function () {
StackExchange.MarkdownEditor.creationCallbacks.add(function (editor, postfix) {
StackExchange.mathjaxEditing.prepareWmdForMathJax(editor, postfix, [["$", "$"], ["\\(","\\)"]]);
});
});
}, "mathjax-editing");

StackExchange.ready(function() {
var channelOptions = {
tags: "".split(" "),
id: "69"
};
initTagRenderer("".split(" "), "".split(" "), channelOptions);

StackExchange.using("externalEditor", function() {
// Have to fire editor after snippets, if snippets enabled
if (StackExchange.settings.snippets.snippetsEnabled) {
StackExchange.using("snippets", function() {
createEditor();
});
}
else {
createEditor();
}
});

function createEditor() {
StackExchange.prepareEditor({
heartbeatType: 'answer',
autoActivateHeartbeat: false,
convertImagesToLinks: true,
noModals: true,
showLowRepImageUploadWarning: true,
reputationToPostImages: 10,
bindNavPrevention: true,
postfix: "",
imageUploader: {
brandingHtml: "Powered by u003ca class="icon-imgur-white" href="https://imgur.com/"u003eu003c/au003e",
contentPolicyHtml: "User contributions licensed under u003ca href="https://creativecommons.org/licenses/by-sa/3.0/"u003ecc by-sa 3.0 with attribution requiredu003c/au003e u003ca href="https://stackoverflow.com/legal/content-policy"u003e(content policy)u003c/au003e",
allowUrls: true
},
noCode: true, onDemand: true,
discardSelector: ".discard-answer"
,immediatelyShowMarkdownHelp:true
});


}
});














draft saved

draft discarded


















StackExchange.ready(
function () {
StackExchange.openid.initPostLogin('.new-post-login', 'https%3a%2f%2fmath.stackexchange.com%2fquestions%2f3093698%2fcomputing-sum-n-1-infty-frac1n2-1-using-fourier-series%23new-answer', 'question_page');
}
);

Post as a guest















Required, but never shown

























3 Answers
3






active

oldest

votes








3 Answers
3






active

oldest

votes









active

oldest

votes






active

oldest

votes









3












$begingroup$

With the correct coefficients (see comments), you have for $x in]-pi,pi[$:



$$
e^{-x} = frac{2}{pi} sinh(pi) + 2 sinh(pi) sum_{n=1}^{infty}frac{ (-1)^n (nsin (nx) + cos(nx)) }{pi(n^2 + 1)}
$$



so



$$
e^{x} + e^{-x} = frac{4}{pi} sinh(pi) + 4 sinh(pi) sum_{n=1}^{infty}frac{ (-1)^n cos(nx) }{pi(n^2 + 1)}
$$



Now we can evaluate this for $x to pmpi$. To be more precise (see the answer by @kvantour), the evaluation $x to pmpi$ corresponds to the average of the two values at $pmpi$ which is exactly what the Fourier series converges to at this point of discontinuity.



$$
cosh (pm pi) = frac{2}{pi} sinh(pi)+ 2 sinh(pi) sum_{n=1}^{infty}frac{1 }{pi(n^2 + 1)}
$$



and in turn



$$
sum_{n=1}^{infty}frac{1 }{(n^2 + 1)} = frac{pi coth(pi) -1}{2 }
$$



Then the second question follows:
$$
sum_{- infty}^{+ infty} frac{1}{n^2 + 1} = 1 + 2 sum_{1}^{+ infty} frac{1}{n^2 + 1} = pi coth(pi) simeq 1.0037 ; pi
$$



Comment: for comparison,
$$
int_{- infty}^{+ infty} frac{1}{x^2 + 1} {rm{dx}} = pi
$$






share|cite|improve this answer











$endgroup$









  • 1




    $begingroup$
    You cannot make the statement $xrightarrowpi$ and just know its value. $f(x)$ is discontinuous in that point.
    $endgroup$
    – kvantour
    Jan 31 at 11:52








  • 1




    $begingroup$
    @kvantour Thank you. I have adressed that point and edited my answer accordingly.
    $endgroup$
    – Andreas
    Jan 31 at 15:13






  • 1




    $begingroup$
    Good point. done.
    $endgroup$
    – Andreas
    Jan 31 at 15:25
















3












$begingroup$

With the correct coefficients (see comments), you have for $x in]-pi,pi[$:



$$
e^{-x} = frac{2}{pi} sinh(pi) + 2 sinh(pi) sum_{n=1}^{infty}frac{ (-1)^n (nsin (nx) + cos(nx)) }{pi(n^2 + 1)}
$$



so



$$
e^{x} + e^{-x} = frac{4}{pi} sinh(pi) + 4 sinh(pi) sum_{n=1}^{infty}frac{ (-1)^n cos(nx) }{pi(n^2 + 1)}
$$



Now we can evaluate this for $x to pmpi$. To be more precise (see the answer by @kvantour), the evaluation $x to pmpi$ corresponds to the average of the two values at $pmpi$ which is exactly what the Fourier series converges to at this point of discontinuity.



$$
cosh (pm pi) = frac{2}{pi} sinh(pi)+ 2 sinh(pi) sum_{n=1}^{infty}frac{1 }{pi(n^2 + 1)}
$$



and in turn



$$
sum_{n=1}^{infty}frac{1 }{(n^2 + 1)} = frac{pi coth(pi) -1}{2 }
$$



Then the second question follows:
$$
sum_{- infty}^{+ infty} frac{1}{n^2 + 1} = 1 + 2 sum_{1}^{+ infty} frac{1}{n^2 + 1} = pi coth(pi) simeq 1.0037 ; pi
$$



Comment: for comparison,
$$
int_{- infty}^{+ infty} frac{1}{x^2 + 1} {rm{dx}} = pi
$$






share|cite|improve this answer











$endgroup$









  • 1




    $begingroup$
    You cannot make the statement $xrightarrowpi$ and just know its value. $f(x)$ is discontinuous in that point.
    $endgroup$
    – kvantour
    Jan 31 at 11:52








  • 1




    $begingroup$
    @kvantour Thank you. I have adressed that point and edited my answer accordingly.
    $endgroup$
    – Andreas
    Jan 31 at 15:13






  • 1




    $begingroup$
    Good point. done.
    $endgroup$
    – Andreas
    Jan 31 at 15:25














3












3








3





$begingroup$

With the correct coefficients (see comments), you have for $x in]-pi,pi[$:



$$
e^{-x} = frac{2}{pi} sinh(pi) + 2 sinh(pi) sum_{n=1}^{infty}frac{ (-1)^n (nsin (nx) + cos(nx)) }{pi(n^2 + 1)}
$$



so



$$
e^{x} + e^{-x} = frac{4}{pi} sinh(pi) + 4 sinh(pi) sum_{n=1}^{infty}frac{ (-1)^n cos(nx) }{pi(n^2 + 1)}
$$



Now we can evaluate this for $x to pmpi$. To be more precise (see the answer by @kvantour), the evaluation $x to pmpi$ corresponds to the average of the two values at $pmpi$ which is exactly what the Fourier series converges to at this point of discontinuity.



$$
cosh (pm pi) = frac{2}{pi} sinh(pi)+ 2 sinh(pi) sum_{n=1}^{infty}frac{1 }{pi(n^2 + 1)}
$$



and in turn



$$
sum_{n=1}^{infty}frac{1 }{(n^2 + 1)} = frac{pi coth(pi) -1}{2 }
$$



Then the second question follows:
$$
sum_{- infty}^{+ infty} frac{1}{n^2 + 1} = 1 + 2 sum_{1}^{+ infty} frac{1}{n^2 + 1} = pi coth(pi) simeq 1.0037 ; pi
$$



Comment: for comparison,
$$
int_{- infty}^{+ infty} frac{1}{x^2 + 1} {rm{dx}} = pi
$$






share|cite|improve this answer











$endgroup$



With the correct coefficients (see comments), you have for $x in]-pi,pi[$:



$$
e^{-x} = frac{2}{pi} sinh(pi) + 2 sinh(pi) sum_{n=1}^{infty}frac{ (-1)^n (nsin (nx) + cos(nx)) }{pi(n^2 + 1)}
$$



so



$$
e^{x} + e^{-x} = frac{4}{pi} sinh(pi) + 4 sinh(pi) sum_{n=1}^{infty}frac{ (-1)^n cos(nx) }{pi(n^2 + 1)}
$$



Now we can evaluate this for $x to pmpi$. To be more precise (see the answer by @kvantour), the evaluation $x to pmpi$ corresponds to the average of the two values at $pmpi$ which is exactly what the Fourier series converges to at this point of discontinuity.



$$
cosh (pm pi) = frac{2}{pi} sinh(pi)+ 2 sinh(pi) sum_{n=1}^{infty}frac{1 }{pi(n^2 + 1)}
$$



and in turn



$$
sum_{n=1}^{infty}frac{1 }{(n^2 + 1)} = frac{pi coth(pi) -1}{2 }
$$



Then the second question follows:
$$
sum_{- infty}^{+ infty} frac{1}{n^2 + 1} = 1 + 2 sum_{1}^{+ infty} frac{1}{n^2 + 1} = pi coth(pi) simeq 1.0037 ; pi
$$



Comment: for comparison,
$$
int_{- infty}^{+ infty} frac{1}{x^2 + 1} {rm{dx}} = pi
$$







share|cite|improve this answer














share|cite|improve this answer



share|cite|improve this answer








edited Jan 31 at 15:25

























answered Jan 30 at 17:48









AndreasAndreas

8,4161137




8,4161137








  • 1




    $begingroup$
    You cannot make the statement $xrightarrowpi$ and just know its value. $f(x)$ is discontinuous in that point.
    $endgroup$
    – kvantour
    Jan 31 at 11:52








  • 1




    $begingroup$
    @kvantour Thank you. I have adressed that point and edited my answer accordingly.
    $endgroup$
    – Andreas
    Jan 31 at 15:13






  • 1




    $begingroup$
    Good point. done.
    $endgroup$
    – Andreas
    Jan 31 at 15:25














  • 1




    $begingroup$
    You cannot make the statement $xrightarrowpi$ and just know its value. $f(x)$ is discontinuous in that point.
    $endgroup$
    – kvantour
    Jan 31 at 11:52








  • 1




    $begingroup$
    @kvantour Thank you. I have adressed that point and edited my answer accordingly.
    $endgroup$
    – Andreas
    Jan 31 at 15:13






  • 1




    $begingroup$
    Good point. done.
    $endgroup$
    – Andreas
    Jan 31 at 15:25








1




1




$begingroup$
You cannot make the statement $xrightarrowpi$ and just know its value. $f(x)$ is discontinuous in that point.
$endgroup$
– kvantour
Jan 31 at 11:52






$begingroup$
You cannot make the statement $xrightarrowpi$ and just know its value. $f(x)$ is discontinuous in that point.
$endgroup$
– kvantour
Jan 31 at 11:52






1




1




$begingroup$
@kvantour Thank you. I have adressed that point and edited my answer accordingly.
$endgroup$
– Andreas
Jan 31 at 15:13




$begingroup$
@kvantour Thank you. I have adressed that point and edited my answer accordingly.
$endgroup$
– Andreas
Jan 31 at 15:13




1




1




$begingroup$
Good point. done.
$endgroup$
– Andreas
Jan 31 at 15:25




$begingroup$
Good point. done.
$endgroup$
– Andreas
Jan 31 at 15:25











1












$begingroup$

When you state Using Fourier series, it does not necessarily mean that you have to use the Fourier Cosine and Sine series. You can also write it in exponential form:




Any periodic function $tilde{f}(t)$ with period $2pi$ can be written
as:



$$ f(t) = sum_{n=-infty}^infty c_n,e^{int} $$ with $$
c_n=frac{1}{2pi}int_{-pi}^pi tilde f(t),e^{-int},textrm{d}t
$$

note: we make a distinction between the fourier series $f(t)$ and the periodic function $tilde f(t)$ to indicate that they are different when $tilde f(t)$ is discontiniuous.




When you apply this for $tilde f(t)$, which is periodic over $2pi$, discontinuous and defined in the region $]-pi,pi[$ as $exp(-t)$, you obtain



$$
c_n = frac{sinh(pi)}picdotfrac{(-1)^n,(1-in)}{1+n^2}
$$



giving you:



$$
f(t)=frac{sinh(pi)}pi sum_{n=-infty}^inftyfrac{(-1)^n,(1-in)}{1+n^2},e^{int}
$$



You already notice the familiar part in the sum which is of interest, the problem is the $(-1)^n$. This we can get rid of with the proper choice of $t$. If $t=pmpi$ then $exp(pm inpi)=(-1)^n$. But be advised $f(t)$ is periodic with a period of $2pi$ and is not continuous in the points $t=npi$. The value the Fourier Series will return is:



$$ f(pi)=f(-pi)= frac{sinh(pi)}pi sum_{n=-infty}^inftyfrac{(1-in)}{1+n^2} = frac{sinh(pi)}pi sum_{n=-infty}^inftyfrac{1}{1+n^2}$$



The latter reduction of the sum is straightforward as the imaginary part is zero.



But we cannot use this, as we do not know what $f(pi)$ is since $tilde f(t)$ is discontinuous at these points. Luckily, some smart people solved this conundrum and showed that at a discontinuity, the Fourier series converges to the average of the two values, i.e.



$$ f(pi)=f(-pi)=frac{lim_{trightarrowpi^-}tilde f(t) + lim_{trightarrow pi^+}tilde f(t)}{2} = frac{exp(-pi)+exp(pi)}{2} = cosh(pi)$$



See: Fourier series at discontinuities



So in the end, we have the solution:
$$f(pi)=cosh(pi)=frac{sinh(pi)}pi sum_{n=-infty}^inftyfrac{1}{1+n^2}$$
or



$$bbox[5px,border:2px solid #00A000]{pi coth(pi)=sum_{n=-infty}^inftyfrac{1}{1+n^2}=1+2sum_{n=1}^inftyfrac{1}{1+n^2}}$$



remark: computing $f(0)$ gives directly, without any fuss:



$$bbox[5px,border:2px solid #000000]{frac{pi}{sinh(pi)}=sum_{n=-infty}^inftyfrac{(-1)^n}{1+n^2}}$$






share|cite|improve this answer











$endgroup$









  • 1




    $begingroup$
    I computed the sum using $f(0)$, but I will pay more attention to the points of discontinuity, so thank you so much for your additional important information.
    $endgroup$
    – Zouhair El Yaagoubi
    Jan 31 at 12:11
















1












$begingroup$

When you state Using Fourier series, it does not necessarily mean that you have to use the Fourier Cosine and Sine series. You can also write it in exponential form:




Any periodic function $tilde{f}(t)$ with period $2pi$ can be written
as:



$$ f(t) = sum_{n=-infty}^infty c_n,e^{int} $$ with $$
c_n=frac{1}{2pi}int_{-pi}^pi tilde f(t),e^{-int},textrm{d}t
$$

note: we make a distinction between the fourier series $f(t)$ and the periodic function $tilde f(t)$ to indicate that they are different when $tilde f(t)$ is discontiniuous.




When you apply this for $tilde f(t)$, which is periodic over $2pi$, discontinuous and defined in the region $]-pi,pi[$ as $exp(-t)$, you obtain



$$
c_n = frac{sinh(pi)}picdotfrac{(-1)^n,(1-in)}{1+n^2}
$$



giving you:



$$
f(t)=frac{sinh(pi)}pi sum_{n=-infty}^inftyfrac{(-1)^n,(1-in)}{1+n^2},e^{int}
$$



You already notice the familiar part in the sum which is of interest, the problem is the $(-1)^n$. This we can get rid of with the proper choice of $t$. If $t=pmpi$ then $exp(pm inpi)=(-1)^n$. But be advised $f(t)$ is periodic with a period of $2pi$ and is not continuous in the points $t=npi$. The value the Fourier Series will return is:



$$ f(pi)=f(-pi)= frac{sinh(pi)}pi sum_{n=-infty}^inftyfrac{(1-in)}{1+n^2} = frac{sinh(pi)}pi sum_{n=-infty}^inftyfrac{1}{1+n^2}$$



The latter reduction of the sum is straightforward as the imaginary part is zero.



But we cannot use this, as we do not know what $f(pi)$ is since $tilde f(t)$ is discontinuous at these points. Luckily, some smart people solved this conundrum and showed that at a discontinuity, the Fourier series converges to the average of the two values, i.e.



$$ f(pi)=f(-pi)=frac{lim_{trightarrowpi^-}tilde f(t) + lim_{trightarrow pi^+}tilde f(t)}{2} = frac{exp(-pi)+exp(pi)}{2} = cosh(pi)$$



See: Fourier series at discontinuities



So in the end, we have the solution:
$$f(pi)=cosh(pi)=frac{sinh(pi)}pi sum_{n=-infty}^inftyfrac{1}{1+n^2}$$
or



$$bbox[5px,border:2px solid #00A000]{pi coth(pi)=sum_{n=-infty}^inftyfrac{1}{1+n^2}=1+2sum_{n=1}^inftyfrac{1}{1+n^2}}$$



remark: computing $f(0)$ gives directly, without any fuss:



$$bbox[5px,border:2px solid #000000]{frac{pi}{sinh(pi)}=sum_{n=-infty}^inftyfrac{(-1)^n}{1+n^2}}$$






share|cite|improve this answer











$endgroup$









  • 1




    $begingroup$
    I computed the sum using $f(0)$, but I will pay more attention to the points of discontinuity, so thank you so much for your additional important information.
    $endgroup$
    – Zouhair El Yaagoubi
    Jan 31 at 12:11














1












1








1





$begingroup$

When you state Using Fourier series, it does not necessarily mean that you have to use the Fourier Cosine and Sine series. You can also write it in exponential form:




Any periodic function $tilde{f}(t)$ with period $2pi$ can be written
as:



$$ f(t) = sum_{n=-infty}^infty c_n,e^{int} $$ with $$
c_n=frac{1}{2pi}int_{-pi}^pi tilde f(t),e^{-int},textrm{d}t
$$

note: we make a distinction between the fourier series $f(t)$ and the periodic function $tilde f(t)$ to indicate that they are different when $tilde f(t)$ is discontiniuous.




When you apply this for $tilde f(t)$, which is periodic over $2pi$, discontinuous and defined in the region $]-pi,pi[$ as $exp(-t)$, you obtain



$$
c_n = frac{sinh(pi)}picdotfrac{(-1)^n,(1-in)}{1+n^2}
$$



giving you:



$$
f(t)=frac{sinh(pi)}pi sum_{n=-infty}^inftyfrac{(-1)^n,(1-in)}{1+n^2},e^{int}
$$



You already notice the familiar part in the sum which is of interest, the problem is the $(-1)^n$. This we can get rid of with the proper choice of $t$. If $t=pmpi$ then $exp(pm inpi)=(-1)^n$. But be advised $f(t)$ is periodic with a period of $2pi$ and is not continuous in the points $t=npi$. The value the Fourier Series will return is:



$$ f(pi)=f(-pi)= frac{sinh(pi)}pi sum_{n=-infty}^inftyfrac{(1-in)}{1+n^2} = frac{sinh(pi)}pi sum_{n=-infty}^inftyfrac{1}{1+n^2}$$



The latter reduction of the sum is straightforward as the imaginary part is zero.



But we cannot use this, as we do not know what $f(pi)$ is since $tilde f(t)$ is discontinuous at these points. Luckily, some smart people solved this conundrum and showed that at a discontinuity, the Fourier series converges to the average of the two values, i.e.



$$ f(pi)=f(-pi)=frac{lim_{trightarrowpi^-}tilde f(t) + lim_{trightarrow pi^+}tilde f(t)}{2} = frac{exp(-pi)+exp(pi)}{2} = cosh(pi)$$



See: Fourier series at discontinuities



So in the end, we have the solution:
$$f(pi)=cosh(pi)=frac{sinh(pi)}pi sum_{n=-infty}^inftyfrac{1}{1+n^2}$$
or



$$bbox[5px,border:2px solid #00A000]{pi coth(pi)=sum_{n=-infty}^inftyfrac{1}{1+n^2}=1+2sum_{n=1}^inftyfrac{1}{1+n^2}}$$



remark: computing $f(0)$ gives directly, without any fuss:



$$bbox[5px,border:2px solid #000000]{frac{pi}{sinh(pi)}=sum_{n=-infty}^inftyfrac{(-1)^n}{1+n^2}}$$






share|cite|improve this answer











$endgroup$



When you state Using Fourier series, it does not necessarily mean that you have to use the Fourier Cosine and Sine series. You can also write it in exponential form:




Any periodic function $tilde{f}(t)$ with period $2pi$ can be written
as:



$$ f(t) = sum_{n=-infty}^infty c_n,e^{int} $$ with $$
c_n=frac{1}{2pi}int_{-pi}^pi tilde f(t),e^{-int},textrm{d}t
$$

note: we make a distinction between the fourier series $f(t)$ and the periodic function $tilde f(t)$ to indicate that they are different when $tilde f(t)$ is discontiniuous.




When you apply this for $tilde f(t)$, which is periodic over $2pi$, discontinuous and defined in the region $]-pi,pi[$ as $exp(-t)$, you obtain



$$
c_n = frac{sinh(pi)}picdotfrac{(-1)^n,(1-in)}{1+n^2}
$$



giving you:



$$
f(t)=frac{sinh(pi)}pi sum_{n=-infty}^inftyfrac{(-1)^n,(1-in)}{1+n^2},e^{int}
$$



You already notice the familiar part in the sum which is of interest, the problem is the $(-1)^n$. This we can get rid of with the proper choice of $t$. If $t=pmpi$ then $exp(pm inpi)=(-1)^n$. But be advised $f(t)$ is periodic with a period of $2pi$ and is not continuous in the points $t=npi$. The value the Fourier Series will return is:



$$ f(pi)=f(-pi)= frac{sinh(pi)}pi sum_{n=-infty}^inftyfrac{(1-in)}{1+n^2} = frac{sinh(pi)}pi sum_{n=-infty}^inftyfrac{1}{1+n^2}$$



The latter reduction of the sum is straightforward as the imaginary part is zero.



But we cannot use this, as we do not know what $f(pi)$ is since $tilde f(t)$ is discontinuous at these points. Luckily, some smart people solved this conundrum and showed that at a discontinuity, the Fourier series converges to the average of the two values, i.e.



$$ f(pi)=f(-pi)=frac{lim_{trightarrowpi^-}tilde f(t) + lim_{trightarrow pi^+}tilde f(t)}{2} = frac{exp(-pi)+exp(pi)}{2} = cosh(pi)$$



See: Fourier series at discontinuities



So in the end, we have the solution:
$$f(pi)=cosh(pi)=frac{sinh(pi)}pi sum_{n=-infty}^inftyfrac{1}{1+n^2}$$
or



$$bbox[5px,border:2px solid #00A000]{pi coth(pi)=sum_{n=-infty}^inftyfrac{1}{1+n^2}=1+2sum_{n=1}^inftyfrac{1}{1+n^2}}$$



remark: computing $f(0)$ gives directly, without any fuss:



$$bbox[5px,border:2px solid #000000]{frac{pi}{sinh(pi)}=sum_{n=-infty}^inftyfrac{(-1)^n}{1+n^2}}$$







share|cite|improve this answer














share|cite|improve this answer



share|cite|improve this answer








edited Jan 31 at 15:20

























answered Jan 31 at 11:58









kvantourkvantour

36829




36829








  • 1




    $begingroup$
    I computed the sum using $f(0)$, but I will pay more attention to the points of discontinuity, so thank you so much for your additional important information.
    $endgroup$
    – Zouhair El Yaagoubi
    Jan 31 at 12:11














  • 1




    $begingroup$
    I computed the sum using $f(0)$, but I will pay more attention to the points of discontinuity, so thank you so much for your additional important information.
    $endgroup$
    – Zouhair El Yaagoubi
    Jan 31 at 12:11








1




1




$begingroup$
I computed the sum using $f(0)$, but I will pay more attention to the points of discontinuity, so thank you so much for your additional important information.
$endgroup$
– Zouhair El Yaagoubi
Jan 31 at 12:11




$begingroup$
I computed the sum using $f(0)$, but I will pay more attention to the points of discontinuity, so thank you so much for your additional important information.
$endgroup$
– Zouhair El Yaagoubi
Jan 31 at 12:11











-2












$begingroup$

Try using Parseval's theorem,



$$frac{1}{pi}int_{-pi}^pi |f(x)|^2 dx = frac{a_0}{2} + sum_{n=1}^{infty} a_n^2+b_n^2$$



If it comes out too nasty, then I think your coeffecients might be off.






share|cite|improve this answer









$endgroup$













  • $begingroup$
    I do not see how Parseval's theorem will give me the sum, can you elaborate more? Although, I have checked the coefficient well. They seem to be correct.
    $endgroup$
    – Zouhair El Yaagoubi
    Jan 30 at 16:12
















-2












$begingroup$

Try using Parseval's theorem,



$$frac{1}{pi}int_{-pi}^pi |f(x)|^2 dx = frac{a_0}{2} + sum_{n=1}^{infty} a_n^2+b_n^2$$



If it comes out too nasty, then I think your coeffecients might be off.






share|cite|improve this answer









$endgroup$













  • $begingroup$
    I do not see how Parseval's theorem will give me the sum, can you elaborate more? Although, I have checked the coefficient well. They seem to be correct.
    $endgroup$
    – Zouhair El Yaagoubi
    Jan 30 at 16:12














-2












-2








-2





$begingroup$

Try using Parseval's theorem,



$$frac{1}{pi}int_{-pi}^pi |f(x)|^2 dx = frac{a_0}{2} + sum_{n=1}^{infty} a_n^2+b_n^2$$



If it comes out too nasty, then I think your coeffecients might be off.






share|cite|improve this answer









$endgroup$



Try using Parseval's theorem,



$$frac{1}{pi}int_{-pi}^pi |f(x)|^2 dx = frac{a_0}{2} + sum_{n=1}^{infty} a_n^2+b_n^2$$



If it comes out too nasty, then I think your coeffecients might be off.







share|cite|improve this answer












share|cite|improve this answer



share|cite|improve this answer










answered Jan 30 at 16:08









Nicholas ParrisNicholas Parris

1563




1563












  • $begingroup$
    I do not see how Parseval's theorem will give me the sum, can you elaborate more? Although, I have checked the coefficient well. They seem to be correct.
    $endgroup$
    – Zouhair El Yaagoubi
    Jan 30 at 16:12


















  • $begingroup$
    I do not see how Parseval's theorem will give me the sum, can you elaborate more? Although, I have checked the coefficient well. They seem to be correct.
    $endgroup$
    – Zouhair El Yaagoubi
    Jan 30 at 16:12
















$begingroup$
I do not see how Parseval's theorem will give me the sum, can you elaborate more? Although, I have checked the coefficient well. They seem to be correct.
$endgroup$
– Zouhair El Yaagoubi
Jan 30 at 16:12




$begingroup$
I do not see how Parseval's theorem will give me the sum, can you elaborate more? Although, I have checked the coefficient well. They seem to be correct.
$endgroup$
– Zouhair El Yaagoubi
Jan 30 at 16:12


















draft saved

draft discarded




















































Thanks for contributing an answer to Mathematics Stack Exchange!


  • Please be sure to answer the question. Provide details and share your research!

But avoid



  • Asking for help, clarification, or responding to other answers.

  • Making statements based on opinion; back them up with references or personal experience.


Use MathJax to format equations. MathJax reference.


To learn more, see our tips on writing great answers.




draft saved


draft discarded














StackExchange.ready(
function () {
StackExchange.openid.initPostLogin('.new-post-login', 'https%3a%2f%2fmath.stackexchange.com%2fquestions%2f3093698%2fcomputing-sum-n-1-infty-frac1n2-1-using-fourier-series%23new-answer', 'question_page');
}
);

Post as a guest















Required, but never shown





















































Required, but never shown














Required, but never shown












Required, but never shown







Required, but never shown

































Required, but never shown














Required, but never shown












Required, but never shown







Required, but never shown







Popular posts from this blog

Can a sorcerer learn a 5th-level spell early by creating spell slots using the Font of Magic feature?

ts Property 'filter' does not exist on type '{}'

mat-slide-toggle shouldn't change it's state when I click cancel in confirmation window